Difference between revisions of "1999 USAMO Problems/Problem 4"

(problem and solution)
 
m
Line 1: Line 1:
 
== Problem ==
 
== Problem ==
 
 
Let <math>a_{1}, a_{2}, \dots, a_{n}</math> (<math>n > 3</math>) be real numbers such that
 
Let <math>a_{1}, a_{2}, \dots, a_{n}</math> (<math>n > 3</math>) be real numbers such that
 
<cmath>
 
<cmath>
Line 11: Line 10:
  
 
== Solution ==
 
== Solution ==
 
 
First, suppose all the <math>a_i</math> are positive.  Then
 
First, suppose all the <math>a_i</math> are positive.  Then
 
<cmath> \max(a_1, \dotsc, a_n) \ge \sqrt{\frac{a_1^2 +
 
<cmath> \max(a_1, \dotsc, a_n) \ge \sqrt{\frac{a_1^2 +
Line 29: Line 27:
 
\end{align*} </cmath>
 
\end{align*} </cmath>
 
Since <math>k<n</math>, <math>4(n-k) > 4</math>.  It follows that <math>\max(a_1, \dotsc, a_n) \ge \sqrt{4} = 2</math>, as desired.  <math>\blacksquare</math>
 
Since <math>k<n</math>, <math>4(n-k) > 4</math>.  It follows that <math>\max(a_1, \dotsc, a_n) \ge \sqrt{4} = 2</math>, as desired.  <math>\blacksquare</math>
 
  
 
{{alternate solutions}}
 
{{alternate solutions}}
  
 
== Resources ==
 
== Resources ==
 
 
* [[1999 USAMO Problems]]
 
* [[1999 USAMO Problems]]
 
* [http://www.artofproblemsolving.com/Forum/viewtopic.php?p=63591#63591 Discussion on AoPS/MathLinks]
 
* [http://www.artofproblemsolving.com/Forum/viewtopic.php?p=63591#63591 Discussion on AoPS/MathLinks]
 
  
 
[[Category:Olympiad Algebra Problems]]
 
[[Category:Olympiad Algebra Problems]]

Revision as of 11:30, 24 April 2008

Problem

Let $a_{1}, a_{2}, \dots, a_{n}$ ($n > 3$) be real numbers such that \[a_{1} + a_{2} + \cdots + a_{n} \geq n \qquad \mbox{and} \qquad a_{1}^{2} + a_{2}^{2} + \cdots + a_{n}^{2} \geq n^{2}.\] Prove that $\max(a_{1}, a_{2}, \dots, a_{n}) \geq 2$.

Solution

First, suppose all the $a_i$ are positive. Then \[\max(a_1, \dotsc, a_n) \ge \sqrt{\frac{a_1^2 + \dotsb + a_n^2}{n}} \ge \sqrt{n} \ge 2 .\] Suppose, on the other hand, that without loss of generality, \[a_1 \ge a_2 \ge \dotsb \ge a_k \ge 0 > a_{k+1} \ge \dotsb \ge a_n,\] with $1\le k <n$. If $a_1 >2$ we are done, so suppose that $a_1 \le2$. Then $\sum_{i=1}^k a_i \le 2k$, so \[\sum_{i=k+1}^n -a_i \le 2k-n .\] Since $-a_i$ is a positive real for all $k+1 \le i \le n$, it follows that

\[\sum_{i=k+1}^n a_i^2 \le \left( \sum_{i=k+1}^n} -a_i \right)^2 \le (2k-n)^2 .\] (Error compiling LaTeX. Unknown error_msg)

Then \begin{align*} \max(a_1, \dotsc, a_n)^2 &\ge \sum_{i=1}^k a_i^2 /k \\ &\ge \left( n^2 - \sum_{i=k+1}^n a_i^2 \right)/k \\ &\ge \frac{n^2 - (2k-n)^2}{k} = 4(n-k). \end{align*} Since $k<n$, $4(n-k) > 4$. It follows that $\max(a_1, \dotsc, a_n) \ge \sqrt{4} = 2$, as desired. $\blacksquare$

Alternate solutions are always welcome. If you have a different, elegant solution to this problem, please add it to this page.

Resources